Đến nội dung

tuan101293 nội dung

Có 316 mục bởi tuan101293 (Tìm giới hạn từ 29-04-2020)



Sắp theo                Sắp xếp  

#275278 Number theory Marathon

Đã gửi bởi tuan101293 on 05-09-2011 - 12:01 trong Số học

anh hiểu nhầm ý em rùi vì y thuộc Z nên $ \sqrt{1-4k^4y^2}$ là ước của 1 (ước của 1 gồm 1 và -1(loại)) từ đó em mới giải đây là cách giải của em nên em cũng sợ sai lắm (bài này trích ở đâu zậy mấy anh để em xem đáp án)

ô hay,
$\sqrt{1-4k^4y^2}$ có nguyên đâu, chỉ hữu tỉ thôi nên nó fai có dạng 1/n



#273372 Number theory Marathon

Đã gửi bởi tuan101293 on 21-08-2011 - 11:06 trong Số học

cho em giải bằng cách này được không ? cho em ý kiến nhé!!
vì x,y là nghiệm nguyên nên ta có: x=ky ( y thuộc Z , k thuộc R)
thay vào PT ta có:
$ 2k^4y^4+1=y^2 \Leftrightarrow y^2(1-2k^4y^2)=1 \Leftrightarrow y= \dfrac{1}{ \sqrt{1-2k^4y^2} } $
vì y thuộc Z nên $ \dfrac{1}{ \sqrt{1-2k^4y^2} } $ cũng thuộc Z
từ đó ta sẽ có $ \sqrt{1-2k^4y^2} =1 \Leftrightarrow k=0 hay y=0 $

chú giải sai chỗ này : vì y thuộc Z nên cái căn kia có dạng 1/n chứ ko fai là 1
***
bài này thì xài xuống thang thôi
China TST 1993
http://www.artofprob...dc926aa#p269097



#272897 Số nguyên tố

Đã gửi bởi tuan101293 on 17-08-2011 - 23:06 trong Số học

sao $p|q-1$ đựoc nhỉ?

mình đánh nhầm p với q
p là cấp của m modulo q nên p|q-1
********
chú ý ở đầu ta xài bổ đề
$(\dfrac{a^n-b^n}{a-b},a-b)=(a-b,n)$



#272846 Số nguyên tố

Đã gửi bởi tuan101293 on 17-08-2011 - 19:51 trong Số học

Đề bài: Chứng minh có vô số số nguyên tố có dạng 2pn +1 với p là số nguyên tố lẻ bất kì.

Xét với số p tùy ý thuộc P, lấy m tùy ý mà (m-1) ko chia hết cho p
xét q thuộc P mà $q|\dfrac{m^p-1}{m-1}$
ta có $(\dfrac{m^p-1}{m-1},m-1)=1$ nên p là cấp của m modulo q hay
$p|q-1$ hay $q=k_m*p+1$
đpcm



#272548 Bài Toán Đẹp

Đã gửi bởi tuan101293 on 15-08-2011 - 20:12 trong Tổ hợp và rời rạc

xét dãy $p_n\in P$ là dãy các số nguyên tố tăng mà $p_n\equiv 1(mod 4)$ (dãy này vô hạn)
hiển nhiên mọi ước số của $p_n^2+1$ cũng thuộc dãy $p_n$ (mọi ước nguyên tố của $x^2+1$ ko có dạng 4k+3)
giả sử ta đã chọn đến $a_{n-1},b_{n-1}$ (luôn chọn dc số đầu,VD $a_1=1,b_1=1$
chọn $a_n=p_k^2+1$ với k đủ lớn để $a_n>b_{n-1}$
và ta chú ý rằng -1 là SCP mod $p_n$ với mọi n
ta phân tích $a_n*(a_n+1)$ ra thành tích các thừa số nguyên tố (toàn các số thuộc dãy $p_n$)

chú ý -1 là SCP mod p thì -1 là SCP mod $p^k$ với k thuộc N
nên -1 là SCP mod $a_n(a_n+1)$ hay tồn tại $b_n$ để $a_n(a_n+1)|b_n^2+1$
hiển nhiên $b_n>a_n>b_{n-1}$
suy ra ta chọn được $a_n,b_n$ tm
theo quy nạp suy ra đPCM



#272543 Một bài vui vui

Đã gửi bởi tuan101293 on 15-08-2011 - 20:01 trong Bất đẳng thức - Cực trị

bài này chính là IRAN 96 (với a,b,c thay bởi ab,bc,ca)



#268286 Cùng nhau thảo luận về IMO 2011

Đã gửi bởi tuan101293 on 12-07-2011 - 21:21 trong Thi HSG Quốc gia và Quốc tế

Nghe từ cục cưng mà cứ thấy làm sao ý
ko bik ý tốt hay là ý j đây



#268284 phương trình hàm

Đã gửi bởi tuan101293 on 12-07-2011 - 21:10 trong Phương trình hàm

IMO 1999
ý tưởng là phương pháp miền giá trị
http://www.artofprob...308b8e3#p131856



#267972 bdt

Đã gửi bởi tuan101293 on 10-07-2011 - 09:57 trong Bất đẳng thức - Cực trị

quy đồng ta phải CM
$\dfrac{\sum a^3(b+c)-2\sum a^2b^2}{4\prod (a+b)}\ge 0$



#267965 cho mình hỏi 2 bài thôi

Đã gửi bởi tuan101293 on 10-07-2011 - 09:28 trong Số học

1) cho $ x_1, x_2, x_3, ..., x_4$ là các số nguyên dương thỏa mãn điều kiện:$x_6=144, x_(n+3)=x_(n+2)*(x_(n+1)+x_n)$, với $n=1. 2, 3, 4.... (1, 2, 3, 4, n, n+1, n+2, n+3 $là các số nằm dưới, không phải lũy thừa). Tính $x_7.$ Bài này mình tìm ra các cặp số từ $x_1$ tới $x_6$ là ${2, 1, 2, 6, 18, 144}$ và ${7, 1, 1, 8, 16, 144}$, nhưng mình không biết các số có được trùng nhau không, và mình cũng chỉ làm mò, không biết cách lý luận chặt chẽ, mọi người kiểm tra và chỉ mình cách chứng minh nhé.
2) hỏi có tồn tại hàm số $f(x)$ thỏa điều kiện:
$f(f(x))=x và f(f(x)+1)=1-x,$ với mọi x thuộc R?. Bài này mình bí hoàn toàn. Mong mọi người xem và giúp mình. Cám ơn mọi người.

Bài 2:
nếu tồn tại a,b để f(a)=f(b) suy ra a=f(f(a))=f(f(b))=b suy ra a=b hay f đơn ánh
f(f(x))=x, thay x bởi (1-x) suy ra f(f(1-x))=1-x suy ra
f(f(x)+1)=f(f(1-x))
f đơn ánh suy ra f(x)+1= f(1-x)
thay x bởi 1-x suy ra f(1-x)+1=f(x)
cộng 2 cái lại suy ra vô lý



#266512 Một bài đại số hay

Đã gửi bởi tuan101293 on 26-06-2011 - 07:37 trong Số học

Chỗ này la sao nhỉ . Mình đọc không hiểu ?

Cái đó là Viet nên có nghiệm $\dfrac{b^2+6}{a}$ và do a nhỏ nhất nên $\dfrac{b^2+6}{a}\ge a\ge b$
nên b nằm ngoài khoảng nghiệm hay $f(b)\ge 0$



#266488 Một bài đại số hay

Đã gửi bởi tuan101293 on 25-06-2011 - 21:03 trong Số học

Ta chỉ cần xét $x,y\in N*$
xét bộ $(a,b)$ thỏa mãn $a\ge b>0$, a nhỏ nhất
đặt $k=\dfrac{x^2+y^2+6}{xy}$
suy ra $x^2-kxy+y^2+6=0 =f(x)$ :D
(a,b) thỏa mãn :D
ngoài ra :Rightarrow còn có bộ nghiệm $(\dfrac{b^2+6}{a},b)$
do a nhỏ nhất nên $(\dfrac{b^2+6}{a})\ge a\ge b$ suy ra $f(b)=b^2(2-k)+6\ge 0$
do $k\ge 3$ suy ra $b^2\le 6$ hay b=1,2
b=1 suy ra k=8 thỏa mãn
b=2 suy ra vô nghiệm
ĐPCM



#266485 Bài đơn giản

Đã gửi bởi tuan101293 on 25-06-2011 - 20:56 trong Số học

Bài Toán :
Tìm số nguyên tố lẻ nhỏ nhất $p$ thoả mãn tính chất :


$ 2^{n+1} | \leftfloor[ (3 + \sqrt{p} )^{2n} \rightfloor] +1 \ \ \forall n \in \mathbb{N}$

Với p=3,thay n=1 ta thấy vô lý
suy ra $p\ge 5$
Ta sẽ CMR với p=5 thì thỏa mãn
thật vậy
do $(3+\sqrt{5})^{2n}+(3-\sqrt{5})^{2n}\in Z_{+}$ và $0<3- \sqrt{5}<1$
suy ra $[(3+\sqrt{5})^{2n}]+1=(3+\sqrt{5})^{2n}+(3-\sqrt{5})^{2n}=x_n$
XD công thức truy hồi $x_{n+1}=28x_{n}-16x_{n-1}$
quy nạp ta có ngay p=5 thỏa mãn



#264503 bất đẳng thức khó

Đã gửi bởi tuan101293 on 12-06-2011 - 08:57 trong Bất đẳng thức - Cực trị

Chứng minh rằng với a,b,c không âm thoả mãn $a+b>0, b+c>0, c+a>0$ ta có:
$\dfrac{a^2}{(ka+b)(ka+c)} + \dfrac{b^2}{(kb+c)(kb+a)} + \dfrac{c^2}{(kc+a)(kc+b)} \le \dfrac{3}{(k+1)^2}$.
Tìm hằng số k nhỏ nhất để BDt luôn đúng. Khi đó đẳng thức xảy ra khi nào.

Cho $a=b,c=0$ (gioi han)
thi ta co $\dfrac{2}{k(k+1)}\le \dfrac{3}{(k+1)^2}$ hay $k\ge 2$
*****
@messi:
TH cua chu lim =1/4 nen bdt dung ???



#264491 Chia hết 7^{n+2}

Đã gửi bởi tuan101293 on 11-06-2011 - 23:06 trong Số học


Bài Toán :

Chứng minh rằng với mọi số nguyên dương $n$ ; ta có :

$ 7^{n+2} | 3^{7^{n}} -2^{7^{n}} -1$
:)

n=1 hiển nhiên đúng
ta CM quy nạp =)):
giả sử :D đúng tới n hay ta có
$3^{7^{n}} \equiv 2^{7^n}+1 (mod 7^{n+2})$
suy ra $3^{7^{n+1}}\equiv (2^{7^{n}}+1)^7 (mod 7^{n+3})$
suy ra $3^{7^{n+1}}-2^{7^{n+1}}-1\equiv 7*2^{7^{n}}*(2^{7^n}+1)(4^{7^{n}}+2^{7^n}+1)^2 (mod 7^{n+3})$
(do $(a+b)^7-a^7-b^7=7ab(a+b)(a^2+ab+b^2)$)
chú ý rằng $(4^{7^n}+2^{7^n}+1)^2=(\dfrac{8^{7^n}-1}{2^{7^n}-1})^2$ và ta có $7^{n+1}|8^{7^n}-1$
suy ra $(4^{7^n}+2^{7^n}+1) \vdots 7^{2n+2} $
hay :icon1: đúng với n+1
theo quy nạp ta có ĐPCM

Supermember :

Chỗ này ; để lời giải đầy đủ ; chí tiết thì cần chứng minh ( hoặc có nêu ra ) :

$ ord_{2} (7^n) = 3.7^{n-1}$



#264375 giai thu xem ?

Đã gửi bởi tuan101293 on 10-06-2011 - 23:11 trong Bất đẳng thức và cực trị

bdt holder
$(1+a^3)(1+b^3)(c^3+1)\ge (c+ab)^3$
làm tương tự 2 cái,nhân lại là ra



#264374 giai giup tui cai ni voi iiiiiiiiiiiiiii

Đã gửi bởi tuan101293 on 10-06-2011 - 23:09 trong Các bài toán Đại số khác

anh xét trên N thôi
$2^x=1+y^z$
suy ra y lẻ
+, nếu z chẵn suy ra $1+y^z\equiv 2 (mod 8)$ hay x=1,y tùy ý ,z=0
+, nếu z lẻ suy ra $2^x=(y+1)(y^{z-1}+...+1)$
hiển nhiên $y^{z-1}+...+1$ là số lẻ suy ra z=1 suy ra $y=2^x-1$ với x tùy ý



#264372 Hàm số không bị chặn

Đã gửi bởi tuan101293 on 10-06-2011 - 23:03 trong Số học

Em chém thử nhé
Xét $k\in P$ đủ lớn (k>a)
xét $p|\dfrac{a^k-1}{a-1}$ mà
$(\dfrac{a^k-1}{a-1},a-1)=(k,a-1)=1$ suy ra $k=ord_a(p)|p-1$
suy ra $p=k*x+1$
với n bất kỳ, theo định lý Trung hoa+ bổ đề Dirichlet ta có tồn tại số nguyên tố k để
$k+1,2k+1,...,nk+1$ là hợp số
sử dụng số k này ta có
$x\ge n+1$ hay $\dfrac{p-1}{k}\ge n+1$ mà $\dfrac{p-1}{k}=\dfrac{p-1}{ord_a(p)}$
nên ta có ĐPCM

Supermember :

Tuấn nói rõ hơn cái đoạn chọn $p$ nhé ; anh không hiểu lắm ; đoạn sau thì hình như đúng :neq
*********
cái đoạn p thì anh chỉ cần xét các n chẵn,ta chọn k sao cho
$2ik+1\equiv 0 (mod p_t)$ với p_t là số nguyên tố >i
nên theo trung hoa thì tồn tại k,kết hợp bổ đề đi rich lê về dãy CSC chứa số nguyên tố ta sẽ tìm được k



#263823 bất đẳng thức khó

Đã gửi bởi tuan101293 on 06-06-2011 - 22:25 trong Bất đẳng thức - Cực trị

ta có
$\dfrac{a^2}{(2a+b)(2b+c)}=\dfrac{a^2}{(2a^2+bc)+2a(a+b+c)}\le \dfrac{1}{9}(\dfrac{a^2}{2a^2+bc}+\dfrac{2a^2}{a(a+b+c)})$
suy ra
$VT\le \dfrac{1}{9}(\sum \dfrac{a^2}{2a^2+bc}+2)$
nên ta chỉ cần CM
$\sum \dfrac{a^2}{2a^2+bc}\le 1$ tương đương $\dfrac{bc}{bc+2a^2}\ge 1$ (đúng theo svac)
ĐPCM



#260790 Một bài bất đẳng thức lớp 9

Đã gửi bởi tuan101293 on 08-05-2011 - 21:16 trong Bất đẳng thức và cực trị

Cho a,b,c la những số không âm . Cm:

$\dfrac{a^3+b^3+c^3}{2abc} + \dfrac{a^2+b^2}{c^2+ab}+ \dfrac{b^2+c^2}{a^2+bc}+\dfrac{c^2+a^2}{b^2+ca} \geq \dfrac{9}{2}$


$VT=\dfrac{a^2}{2bc}+\dfrac{b^2}{2ca}+\dfrac{c^2}{2ab} + \dfrac{a^2}{c^2+ab}+\dfrac{b^2}{c^2+ab}+ \dfrac{b^2}{a^2+bc}+\dfrac{c^2}{a^2+bc}+\dfrac{c^2}{b^2+ca}+\dfrac{a^2}{b^2+ca} \geq \dfrac{(3(a+b+c))^2}{2(a^2+b^2+c^2+2ab+2bc+2ca)}=\dfrac{9}{2} $



#260785 Dãy số

Đã gửi bởi tuan101293 on 08-05-2011 - 21:06 trong Số học

có thể
$a_n=\dfrac{6n+9.(-1)^n+11}{4}$



#260781 Một số bài toán sưu tầm

Đã gửi bởi tuan101293 on 08-05-2011 - 20:59 trong Tài liệu - Đề thi

bài 4:
đặt $a=\dfrac{x+m}{x-1},b=...,c=....$
ta có $x=\dfrac{m+a}{a-1},y=\dfrac{m+b}{b-1},z=...$
vì xyz=1 suy ra $(m+a)(m+b)(m+c)=(a-1)(b-1)(c-1)$ :)
chú ý rằng nếu m=-1 thì hiển nhiên bdt đúng
ta xét TH m khác -1
từ :) suy ra $m^2-m+1+(m-1)\sum a+\sum ab=0$
ta phải Cm $a^2+b^2+c^2\ge 1$ cộng 2 lần đẳng thức trên vào suy ra ta phải CM
$\sum a^2+2\sum ab+2(m-1)\sum a+2(m^2-m+1)\ge 1$ hay là $(\sum a+m-1)+m^2\ge 0$ (đúng )
ĐPCM



#260649 Chứng minh rằng với mọi $a,b,c \geq 1$, ta có: $$...

Đã gửi bởi tuan101293 on 08-05-2011 - 09:13 trong Bất đẳng thức - Cực trị

Chứng minh rằng với mọi $a,b,c \geq 1$, ta có:
$\sqrt{a-1}+\sqrt{b-1}+\sqrt{c-1} \leq \sqrt{c(ab+1)}$
(Olympic Toán Hồng Kông)

Một điều đặt ra ở đây đó là bạn sẽ làm gì đầu tiên với một bài toán có dạng như thế này :)

ta có
$\sqrt{x-1}+\sqrt{y-1}\le \sqrt{xy}$ với mọi $x,y\ge 1$
tương đương $x-1+y-1+2\sqrt{(x-1)(y-1)}\le xy$ hay là $(x-1)(y-1)+1\ge 2\sqrt{(x-1)(y-1)}$ Đúng theo côsi
áp dụng ta có
$\sqrt{a-1}+\sqrt{b-1}+\sqrt{c-1}\le \sqrt{ab}+\sqrt{c-1}=\sqrt{(ab+1)-1}+\sqrt{c-1}\le \sqrt{c(ab+1)}$
đpcm
hình như ko có dấu đẳng thức



#260558 Đề thi vào trường chuyên Lê Quý Đôn

Đã gửi bởi tuan101293 on 07-05-2011 - 18:04 trong Bất đẳng thức - Cực trị

b, Cho $\ a,b,c >0 , a + b+c = 1 $ CMR
$\dfrac{19 b^{3} - a^{3} }{ab + 5 b^{2} } + \dfrac{19 c^{3} - b^{3} }{cb + 5 c^{2}} + \dfrac{19 a^{3} - c^{3} }{ac + 5 a^{2}} \leq 3 $

Đây là bài đã từng được đăng trên THTT rồi mà , bạn nào có thì post lên


$\dfrac{19b^3-a^3}{ab+5b^2}\le 4b-a$
cộng dọc lại ta có đpcm



#259892 Thử sức với BDT khó

Đã gửi bởi tuan101293 on 03-05-2011 - 16:45 trong Bất đẳng thức - Cực trị

may mắn thôi
:delta
đùa đó,thành phương pháp rùi bạn à